Finding X-Coordinates of Parallel Tangent and Secant Lines in y = x^3 Graph

  • Thread starter Thread starter Lord Dark
  • Start date Start date
Click For Summary
To find the x-coordinates where the tangent line to the graph of y = x^3 is parallel to the secant line between x = -1 and x = 1, the correct slope of the secant line is determined to be m = 1, not 3. The derivative of the function, y' = 3x^2, is set equal to this slope, leading to the equation 3x^2 = 1. Solving this gives x = ±sqrt(1/3) as the points where the tangent lines are parallel to the secant line. It is important to recognize that this equation has two solutions, corresponding to two points on the graph. Visualizing the graph can aid in understanding the relationship between the tangent and secant lines.
Lord Dark
Messages
120
Reaction score
0

Homework Statement


hi everyone ,, got this question and I need the idea today if possible:

Find the x-coordinate of the point on the graph of y = x^3 where
the tangent line is parallel to the secant line that cuts the curve at x = -1 and x = 1.

Homework Equations




The Attempt at a Solution


i got the derivative and then the tangent line equaled m=3 and then i got this equation:
y=3x+2 so my x coordinates are (1,5) (-1,-1) ,, and I think it's wrong because (-1,-1) is not parallel ,, so how to get it and then what should i do to get the coordinates ,,
 
Last edited:
Physics news on Phys.org
Nope. Your line has slope m = 3, which is not equal to the slope of the secant line. What are the coordinates of the points on the graph of y = x^3 when x = 1 and when x = -1?

There are two points of the graph of y = x^3 with tangent lines that are parallel to the given secant line.
 
yea,, got it ,, m =1 not 3 ,, and then i should y`=3x^2
3x^2=1 then I'll get the coordinates ,, x=+-sqrt(1/3) ,, thanks :D
 
Keep in mind that 3x^2 = 1 has two solutions.

I hope that you have drawn a graph of the function you're working with...
 
Question: A clock's minute hand has length 4 and its hour hand has length 3. What is the distance between the tips at the moment when it is increasing most rapidly?(Putnam Exam Question) Answer: Making assumption that both the hands moves at constant angular velocities, the answer is ## \sqrt{7} .## But don't you think this assumption is somewhat doubtful and wrong?

Similar threads

  • · Replies 5 ·
Replies
5
Views
1K
  • · Replies 11 ·
Replies
11
Views
2K
Replies
1
Views
1K
  • · Replies 8 ·
Replies
8
Views
2K
Replies
15
Views
2K
  • · Replies 1 ·
Replies
1
Views
971
  • · Replies 2 ·
Replies
2
Views
2K
  • · Replies 11 ·
Replies
11
Views
2K
  • · Replies 9 ·
Replies
9
Views
2K
  • · Replies 3 ·
Replies
3
Views
1K